Questions tagged [calculus]

For basic questions about limits, continuity, derivatives, differentiation, integrals, and their applications, mainly of one-variable functions.

Filter by
Sorted by
Tagged with
4 votes
1 answer
393 views

Does this series violate the integral test condition for convergence/divergence?

My professor is asking me to determine if the following series is convergent or divergent using the integral test specifically. $$ \sum^{\infty}_{n=1}n^2e^{-n} $$ However, this seems to violate the ...
Ronan Howard's user avatar
1 vote
1 answer
43 views

Is there a nice closed form of the following function: $f(k) = \lim_{n \rightarrow \infty}\prod^{kn}_{i = 0}\frac{n-2i+1}{n-2i}$

I am tring to find the closed from of the following function: $$f(k) = \lim_{n \rightarrow \infty}\prod^{kn}_{i = 0}\frac{n-2i+3}{n-2i+2},$$ where $k \in [0,\frac{1}{2}]$ If the numerator is $n-2i+4$ ...
0099ax43's user avatar
-2 votes
0 answers
32 views

"Check Stokes' theorem in the plane where $f(x, y) = y^2\mathbf{i} + x^2\mathbf{j}$ and the region formed is bounded by the circle ($x^2+y^2=4$)."

Resolution "The question is a problem from Leithold's calculus book. I didn't understand the ($x = 5 \cos(t)$). Shouldn't it be ($x = 2 \cos(t)$)? Resolution The theorem in the book is this. From ...
lina rey's user avatar
0 votes
1 answer
33 views

What is the significance of defining the partial derivative as a one-sided limit or a two-sided limit?

Let $f:\mathbb{R}^n\to\mathbb{R}$ be a function. We can define the partial derivative of f with respect to its first argument using either a two-sided limit ($h\to 0$) $$\lim_{h\to 0} \frac{f(x_1+h,...
Jürgen Sukumaran's user avatar
0 votes
0 answers
18 views

Critical points and Points of Extremum

Wikipedia says that: By Fermat's theorem, all local maxima and minima of a continuous function occur at critical points But I think that all local maxima and minima, irrespective of whether the ...
Sid N's user avatar
  • 68
-1 votes
0 answers
23 views

How to calculate the ranking of metrices [closed]

In the image I have list of metrices against two accounts. I need to create a mechanism to calculate the ranking of metrices. The condition is that some metrices are good on lower side (like attrition)...
Rahul Mangla's user avatar
0 votes
4 answers
79 views

What is the maximum and minimum value of the following function?

I came across a question in the book Calculus for the Practical Man. The question stated to find the maximum or minimum value of the following function: $$f(x) = \frac{1}{4}\cos^2x - \sin 2x$$ I tried ...
Rishith Raj Raizada's user avatar
-1 votes
0 answers
21 views

Essential Range of a measurable function [closed]

im trying to come up with a solution to the next problem, but i dont know how to handle it. Im relative new to $L^p$ spaces and i would appreciate any help. Thanks in advance! "Given ($\Omega,\...
Iván's user avatar
  • 1
4 votes
2 answers
63 views

Showing divergence of $\sum\limits_{k=1}^{\infty} \log\left(1+\frac{(-1)^{k+1}}{k^\alpha}\right)$ where $0<\alpha<\frac{1}{2}$

I am trying to prove that $$\sum\limits_{k=1}^{\infty} \log\left(1+\frac{(-1)^{k+1}}{k^\alpha}\right)$$ diverges, for any $0<\alpha<\frac{1}{2}$. I tried showing this by taking the Taylor ...
Ofek Levy's user avatar
0 votes
0 answers
35 views

Evaluating the limit of a function involving exponential and hypergeometric functions as it approaches infinity.

I am trying to evaluate the limit of $$ e^{-BY} \left[ B \,e^{2Y} {}_2F_1 \left( 1,1-\frac{B}{2}; 2-\frac{B}{2}; -e^{-2Y} \right) -(B-2)\, {}_2F_1 \left( 1,-\frac{B}{2}; 1-\frac{B}{2}; -e^{-2Y} \...
keynes's user avatar
  • 405
5 votes
4 answers
196 views

Can I avoid differentiation to find the remainder when $x^{73}-2x^{15}+3x-1$ is divided by $(x-1)^2$

The problem states: Find the remainder when $x^{73}-2x^{15}+3x-1$ is divided by $(x-1)^2$. So, what I did is, Assume, the remainder to be linear, i.e $r(x) = ax+b$ By Eucild''s, $x^{73}-2x^{15}+3x-1 ...
Krave37's user avatar
  • 455
-2 votes
0 answers
23 views

Derivative of Gamma and partial sums [closed]

I need to prove that $$ S_N(x)=\sum_{j=1}^N{\frac{1}{j-x}}=\Psi(N+1-x)-\Psi(1-x) $$ Any help is welcome, thanks.
user138364's user avatar
0 votes
1 answer
28 views

finding work required to pump water out of a tank

The tank has the shape of a horizontal cylinder with radius $r$ and length $l$. The water exists through a small opening on the top right side of the cylinder. It seems that there are two ways to ...
rudytheduck's user avatar
-1 votes
0 answers
37 views

$y''−3y' +2y = 4t+ e^(2t)$ where $y(0)= 1$ and $y'(0) = −1$. [closed]

solve the following ODE through laplace transform and then apply partial fraction then use inverse laplce transform.
Rachit Arun's user avatar
1 vote
1 answer
38 views

Let 𝑓:(𝑎,𝑏)→ℝ be a differentiable function. Show that if 𝑓(𝑐)=sup{𝑓(𝑥)∣𝑥∈(𝑎,𝑏)} for some 𝑐∈(𝑎,𝑏) , then 𝑓′(𝑐)=0 .

Here is what I have so far: suppose f'(c) > 0 then lim (f(x) - f(c)) / (x - c) = f'(c) > 0 there exists some delta > 0 s.t. if abs(x-c) < delta then lim (f(x) - f(c)) / (x - c) > 0 take ...
Expln_fortnite_terms's user avatar
0 votes
0 answers
43 views

In what sense are differentials from calculus covectors in differential geometry? Why would there be a covector present in integration?

To preface, I am currently following eigenchris' series on tensor calculus in an effort to learn GR. I believe I understand the sense in which differential operators representing derivatives are ...
Oobleck's user avatar
3 votes
1 answer
79 views
+50

The positive Laplacian is indeed the negative Laplacian

I know this question sounds like a joke. And it probably is:). I found it kind of annoying, but also interesting, to call $-\Delta=-\sum_{j=1}^n\partial^2_{jj}$ "the positive Laplacian" as ...
Liding Yao's user avatar
  • 1,444
0 votes
0 answers
31 views

When is a particle speeding up/down based on position graph

Question: Based on this position graph of a particle, how am I supposed to know when it is speeding up or slowing down? I know that with a velocity graph, when velocity and acceleration have the same ...
mrD's user avatar
  • 1
0 votes
2 answers
88 views

Using triple integration for the area of a triangle

For practice I am trying to use the standard definition of volume for a set $X \subset \mathbb{R}^n$ given by: $$vol(X) := \int_X dx_1 \ \ldots \ dx_n$$ to compute the area of the triangle spanned by ...
3nondatur's user avatar
  • 4,182
0 votes
0 answers
36 views

What is the derivative of x with respect to 2x? [closed]

What is $$\frac{d}{d(2x)}(x)$$ Is it $\frac{1}{2}$ since setting $y=2x$, and rewriting in terms of y $$\frac{d}{dy}(\frac{1}{2}y) = \frac{1}{2}$$ or is it not defined?
Tomek Dobrzynski's user avatar
2 votes
0 answers
19 views

Negative $r$ in polar coordinate while integrating [duplicate]

The question asks $\iint_R (3x+4y^2)\; dA$ where $R$ is the region in the upper half plane bounded by the circles $x^2+y^2=1$ and $x^2+y^2=4$ $$\int_0^\pi \int_1^2 (3r\cos \theta + 3r^2 \sin^2 \theta) ...
IrbidMath's user avatar
  • 3,187
2 votes
3 answers
82 views

Why are these primitives containing $\arcsin x$ equal up to a constant?

While trying to solve $\displaystyle\int\sqrt{14x-x^2}\;dx$, I obtained three different primitives in three different ways: Method 1: completing the square $c(x)=\dfrac{1}{2}\left[49\arcsin\left(\...
orion2112's user avatar
  • 594
6 votes
2 answers
125 views

Seeking clarification: Convergence of the series $\sum_{n=1}^{\infty} \frac{1}{e^{n^2}}$

I was asked to show whether the following series converges: $$\sum_{n=1}^{\infty}\frac{1}{{e^{n^2}}}$$ Here's my first attempt which I thought was pretty straightforward: $$e^{n^{2}}=(e^{n})^{n}\ge e^{...
bwootton's user avatar
  • 125
4 votes
2 answers
117 views

Prove that $\lim_ {x\to \infty}\min_{m,n \in \mathbb{Z}}|x-\sqrt{m^2+2n^2}|=0$ [closed]

For $x\gt0$, let $f(x)$ be the minimum value of $|x-\sqrt{m^2+2n^2}|$ over all integers $m, n$. Then Prove that $$\lim_ {x\to \infty} f(x)= \; 0$$ I am completely stuck with this question. If I take $\...
Skdmg's user avatar
  • 527
0 votes
0 answers
34 views

Is this right for converting an infinite sum into an integral with the limit $[0; \infty]$?

I've done the following to get from an infinite sum to an integral, the question is now, did i do everything properly and didn't made any mistakes, because i coulnd't find anything like that when ...
octodino's user avatar
0 votes
0 answers
37 views

Washer method around X-axis vs other axis

I had a question I was hoping someone could answer. I'm working on this homework problem, and when graphed, the volume looks like it would be the same revolving around the x-axis or on the axis of $y=...
onKtun's user avatar
  • 1
0 votes
1 answer
19 views

Analysis of Metric Properties in an Infinite Set with Discrete Metric

I am not sure if my solution to the following problem is correct Let x be an infinite set. for x $\in$ X and y $\in$ X we define: \begin{equation} d(x,y) = \left\{ \begin{array}{ll} 1 & x \...
kzaf's user avatar
  • 21
0 votes
1 answer
55 views

Understanding the limit of a function

I've always heard that if the value of a function approaches a given limit as the value of the argument approaches a specific value, then the limit exists. But here is an example that has me confused; ...
Christopher Weronski's user avatar
0 votes
0 answers
18 views

How can one compute rounding preservant integrable functions?

Background & Context : The background of the question is an engineering problem. I want to efficiently represent a set of integers as rounded real valued functions and quickly be able to calculate ...
mathreadler's user avatar
  • 25.8k
0 votes
0 answers
29 views

Can billiards fill any figure?

For any 2D closed shape, is there always a point and angle for which the billiard movement of a ball fully completes the interior of the shape? In other words, does the path eventually converge to the ...
sam wolfe's user avatar
  • 3,335
1 vote
1 answer
81 views

$f(x)$ is continuous on $[a, b]$ and assumes one single critical point at $x = c$ that is a local minimum. Prove that $f(c)$ is the absolute minimum.

I was working on a textbook problem yesterday when I found myself needing to prove a statement to make my main argument more rigorous. The statement seems relatively simple at first, yet I ended up ...
ten_to_tenth's user avatar
-3 votes
1 answer
85 views

Calculation of $\lim_{n \to \infty}\frac{ 1^n + 2 ^n + \dots + n^n}{n^n}$ in two different ways giving different answers [closed]

I calculated $ L= \lim_{n \to \infty} \frac {1^n + 2 ^n + \cdots + n^n }{n^n}$ in two different ways getting two different answers. $$L = \lim_{n \to \infty}\left[1 + (\frac {n-1}{n})^n + (\frac {n-...
Anirban 's user avatar
1 vote
1 answer
83 views

Transformation of a random variable vs. joint transformation of several random variables

Let $X \sim f_X(x)$ and $Y = g(X)$. If $g(X)$ is a differentiable, monotonic function with inverse such that $X = g^{-1}(Y)$ then the PDF of $Y$ can be described: $$ f_Y(y) = f_X(g^{-1}(y)) \bigg| \...
Joseph's user avatar
  • 361
1 vote
2 answers
114 views

An integral question which i have never encountered before

I = $\int _{0}^{3}\left( 1+x^{2}\right) d[ x]$ $a)12$ $b)17$ $c)15$ $d)19$ where $[x]$ is the greatest integer less than or equal to $x$, shouldn't this integral be $0$ since $d[x]$ is 0 for all $x$ ...
Aryan Pandey's user avatar
3 votes
1 answer
29 views

Inconsistent Function Monotonicity from hand and Mathematica image

$g(x)=\frac{\phi(x)}{1-\Phi(x)}$, where $\phi(x)$ and $\Phi(x)$ are p.d.f and c.d.f of standard normal distribution respectively. $g'(x)=\frac{\phi'(x)(1-\Phi(x))+\phi^2(x)}{(1-\Phi(x))^2}=\frac{\phi(...
Kozack51's user avatar
1 vote
0 answers
49 views

Residue theorem integral - calculating with trigonometric functions

I am trying to solve the following integral: $$I = \int_0^{2\pi} \frac{d\phi}{a + 2 b \cos(\phi) + 2 c \sin(\phi)}$$ We can assume that the denominator is always strictly positive ($a \gg b, c$). ...
Manuel Ballester's user avatar
1 vote
0 answers
49 views

Difficult Vectors Problem (Calculus & Vectors 12)

Find parametric equations of a line that intersects line 1 and line 2 at right angles. Line 1: $[x,y,z] = [4,8,-1] + t[2,3,-4]$ and Line 2: $[x,y,z] = [7,2,-1] + k[-6,1,2]$. I've tried solving this ...
math's user avatar
  • 11
0 votes
1 answer
37 views

Quantifiers uniform continuity

According to this answer: https://math.stackexchange.com/a/2582334/1098426 We know $\forall x \ \exists y \ \forall z$ differs from $\forall x \forall z\ \exists y$ insofar as $y$ depends on $x$ ...
isaac's user avatar
  • 193
0 votes
0 answers
25 views

Maximizing a function for $k \in [-\pi, \pi]$

I want to find the maximum of $|g(\kappa)|$ defined as follows For some $A>0$, $\alpha\in (0,\infty)$, and $\kappa \in [-\kappa^{*},\kappa^{*}]\subset [-\pi,\pi]$, define $g(\kappa )$ \begin{align}...
KZ-Spectra's user avatar
1 vote
0 answers
58 views

Show that there exists $L \in \mathbb{R}_+^*$ such that for all $x \in [a,b]$ $|g(x) - c| \leq L|x - c|^2$

I encountered an exercise from the admission exam for the Moroccan preparatory class Al-Zahrawi where I faced a small problem with question 4. I was able to answer the question, but the statement ...
Nour Belghazi's user avatar
0 votes
4 answers
81 views

What is wrong about this seemingly simple false proof? [duplicate]

It came accros my mind when doing an exercise in calculus. Consider the following inequalities: $1\leq 2\leq 5$ $1\leq 3\leq 5$ Adding the equaions up is totally fine. But subtracting them, we get: $0\...
natitati's user avatar
4 votes
3 answers
117 views

How to approach an Hyperbolic Integral that doesn't appear to be solvable in closed form.

I'm interested in tackling the following integral: $$\int_{-\ln (2+\sqrt 5)}^{\ln (2+\sqrt 5)} \sqrt{4+\sinh^2(x)} dx$$ While I've attempted various techniques, it appears challenging to find a closed-...
Mark's user avatar
  • 7,839
1 vote
2 answers
194 views

Compute $\int \frac{\sin(x)\cos(x)}{x^2+1}dx$

Compute $$\int \frac{\sin(x)\cos(x)}{x^2+1}dx$$ Attempt $u=\sin(x)\Rightarrow du=\cos(x)dx$ $$\int\frac{u}{x^2+1}du$$ But, now I have both $u$ and $x$ in the function. How do I resolve? Second attempt:...
nirates biadenroc's user avatar
3 votes
2 answers
397 views

Are there two contradictory definitions of the limit of a function at a point?

I have found two different definitions of the limit of a function f at a point a in very popular calculus/analysis texts. In one, the function f is asked to be defined on an open interval containing ...
Guille's user avatar
  • 33
-3 votes
0 answers
27 views

Shouldn't it be like that when Lim ∆x→0 Tan Ψ=tan ∅ [closed]

see the highlighted portion and please try to clear my doubt. $$\tan \psi = \lim_{\Delta x \to 0} \tan \theta$$
aaditya's user avatar
-1 votes
0 answers
47 views

Using double integrals to find the center of mass [closed]

Let $f(x,y)$ be the density of mass in the region; $$R:\begin{cases} 0<x<1\\ 0<y<\sqrt{(1-x^2)}\\ \end{cases}$$ Find the center of mass and the moments of inertia $I_x$, $...
Temiloluwa Adeola's user avatar
1 vote
1 answer
40 views

Expectation of X^2 using complement of CDF

For a nonnegative random variable $$E[X]=\int_0^\infty P(X >x)dx$$ This video on a question from IIT JAM 2023 extends it to $$E[X^2]=\int_0^\infty 2x P(X >x)dx$$ How was the result was arrived ...
Starlight's user avatar
  • 1,670
0 votes
3 answers
83 views

Confusion on finding if a limit exists or not.

I have started the introduction of calculus in high school, and I am confused about this problem. Define $f(x) = -x$ for $x<0$. Then find out if $\lim_{x \to 0} f(x)$ exists or not. If it does, ...
Hayst's user avatar
  • 172
-1 votes
0 answers
26 views

That's right ? $\operatorname{Re}(\sqrt{-z}\delta)>0~$

Helle, everyone. I want to check this question and find it in an article. so that $$s(r_{0},\theta_{0})~:=\left\{z\in\mathbb{C}\setminus\{0\}:|z|\geqslant r_{0}~~\text{ et}~~|\arg\left(z\right)\bigr|...
Ellen Ellen's user avatar
  • 2,319
1 vote
2 answers
119 views

Generalised integral $\int_{0}^{\infty} \int_{0}^{y} \sqrt{x^2 + y^2} e^{-x^2 - y^2} dx dy$

I am having trouble understanding how to compute this integral $$\int_{0}^{\infty} \int_{0}^{y} \sqrt{x^2 + y^2} e^{-x^2 - y^2} \, dx \, dy$$ My idea is to consider the whole plane xy plane instead of ...
Teodoras Paura's user avatar